logarithmic transformation from exponential to linear equation












0














How to convert this exponential equation to linear equation.



Y =exp(17.9348)*x^(-2.705)



what I did is:
Y =log(17.9348)+(-2.705)*log(x)



I am confused with this one.
Y=17.9348+(-2.705)*log(x)



which one is correct transformation?



Thanks in advance!










share|cite|improve this question







New contributor




user3063 is a new contributor to this site. Take care in asking for clarification, commenting, and answering.
Check out our Code of Conduct.

























    0














    How to convert this exponential equation to linear equation.



    Y =exp(17.9348)*x^(-2.705)



    what I did is:
    Y =log(17.9348)+(-2.705)*log(x)



    I am confused with this one.
    Y=17.9348+(-2.705)*log(x)



    which one is correct transformation?



    Thanks in advance!










    share|cite|improve this question







    New contributor




    user3063 is a new contributor to this site. Take care in asking for clarification, commenting, and answering.
    Check out our Code of Conduct.























      0












      0








      0







      How to convert this exponential equation to linear equation.



      Y =exp(17.9348)*x^(-2.705)



      what I did is:
      Y =log(17.9348)+(-2.705)*log(x)



      I am confused with this one.
      Y=17.9348+(-2.705)*log(x)



      which one is correct transformation?



      Thanks in advance!










      share|cite|improve this question







      New contributor




      user3063 is a new contributor to this site. Take care in asking for clarification, commenting, and answering.
      Check out our Code of Conduct.











      How to convert this exponential equation to linear equation.



      Y =exp(17.9348)*x^(-2.705)



      what I did is:
      Y =log(17.9348)+(-2.705)*log(x)



      I am confused with this one.
      Y=17.9348+(-2.705)*log(x)



      which one is correct transformation?



      Thanks in advance!







      logarithms linear-transformations transformation






      share|cite|improve this question







      New contributor




      user3063 is a new contributor to this site. Take care in asking for clarification, commenting, and answering.
      Check out our Code of Conduct.











      share|cite|improve this question







      New contributor




      user3063 is a new contributor to this site. Take care in asking for clarification, commenting, and answering.
      Check out our Code of Conduct.









      share|cite|improve this question




      share|cite|improve this question






      New contributor




      user3063 is a new contributor to this site. Take care in asking for clarification, commenting, and answering.
      Check out our Code of Conduct.









      asked 2 days ago









      user3063

      1071




      1071




      New contributor




      user3063 is a new contributor to this site. Take care in asking for clarification, commenting, and answering.
      Check out our Code of Conduct.





      New contributor





      user3063 is a new contributor to this site. Take care in asking for clarification, commenting, and answering.
      Check out our Code of Conduct.






      user3063 is a new contributor to this site. Take care in asking for clarification, commenting, and answering.
      Check out our Code of Conduct.






















          1 Answer
          1






          active

          oldest

          votes


















          1














          The answer is neither! You need to take the logarithm of both sides:
          begin{align*}
          y&=e^{17.9348},x^{-2.705} \
          ln(y)&=lnleft(e^{17.9348},x^{-2.705}right) \
          ln(y)&=lnleft(e^{17.9348}right)+lnleft(x^{-2.705}right) \
          ln(y)&=17.9348-2.705ln(x).
          end{align*}






          share|cite|improve this answer





















          • ok, thank you so much!
            – user3063
            2 days ago











          Your Answer





          StackExchange.ifUsing("editor", function () {
          return StackExchange.using("mathjaxEditing", function () {
          StackExchange.MarkdownEditor.creationCallbacks.add(function (editor, postfix) {
          StackExchange.mathjaxEditing.prepareWmdForMathJax(editor, postfix, [["$", "$"], ["\\(","\\)"]]);
          });
          });
          }, "mathjax-editing");

          StackExchange.ready(function() {
          var channelOptions = {
          tags: "".split(" "),
          id: "69"
          };
          initTagRenderer("".split(" "), "".split(" "), channelOptions);

          StackExchange.using("externalEditor", function() {
          // Have to fire editor after snippets, if snippets enabled
          if (StackExchange.settings.snippets.snippetsEnabled) {
          StackExchange.using("snippets", function() {
          createEditor();
          });
          }
          else {
          createEditor();
          }
          });

          function createEditor() {
          StackExchange.prepareEditor({
          heartbeatType: 'answer',
          autoActivateHeartbeat: false,
          convertImagesToLinks: true,
          noModals: true,
          showLowRepImageUploadWarning: true,
          reputationToPostImages: 10,
          bindNavPrevention: true,
          postfix: "",
          imageUploader: {
          brandingHtml: "Powered by u003ca class="icon-imgur-white" href="https://imgur.com/"u003eu003c/au003e",
          contentPolicyHtml: "User contributions licensed under u003ca href="https://creativecommons.org/licenses/by-sa/3.0/"u003ecc by-sa 3.0 with attribution requiredu003c/au003e u003ca href="https://stackoverflow.com/legal/content-policy"u003e(content policy)u003c/au003e",
          allowUrls: true
          },
          noCode: true, onDemand: true,
          discardSelector: ".discard-answer"
          ,immediatelyShowMarkdownHelp:true
          });


          }
          });






          user3063 is a new contributor. Be nice, and check out our Code of Conduct.










          draft saved

          draft discarded


















          StackExchange.ready(
          function () {
          StackExchange.openid.initPostLogin('.new-post-login', 'https%3a%2f%2fmath.stackexchange.com%2fquestions%2f3060688%2flogarithmic-transformation-from-exponential-to-linear-equation%23new-answer', 'question_page');
          }
          );

          Post as a guest















          Required, but never shown

























          1 Answer
          1






          active

          oldest

          votes








          1 Answer
          1






          active

          oldest

          votes









          active

          oldest

          votes






          active

          oldest

          votes









          1














          The answer is neither! You need to take the logarithm of both sides:
          begin{align*}
          y&=e^{17.9348},x^{-2.705} \
          ln(y)&=lnleft(e^{17.9348},x^{-2.705}right) \
          ln(y)&=lnleft(e^{17.9348}right)+lnleft(x^{-2.705}right) \
          ln(y)&=17.9348-2.705ln(x).
          end{align*}






          share|cite|improve this answer





















          • ok, thank you so much!
            – user3063
            2 days ago
















          1














          The answer is neither! You need to take the logarithm of both sides:
          begin{align*}
          y&=e^{17.9348},x^{-2.705} \
          ln(y)&=lnleft(e^{17.9348},x^{-2.705}right) \
          ln(y)&=lnleft(e^{17.9348}right)+lnleft(x^{-2.705}right) \
          ln(y)&=17.9348-2.705ln(x).
          end{align*}






          share|cite|improve this answer





















          • ok, thank you so much!
            – user3063
            2 days ago














          1












          1








          1






          The answer is neither! You need to take the logarithm of both sides:
          begin{align*}
          y&=e^{17.9348},x^{-2.705} \
          ln(y)&=lnleft(e^{17.9348},x^{-2.705}right) \
          ln(y)&=lnleft(e^{17.9348}right)+lnleft(x^{-2.705}right) \
          ln(y)&=17.9348-2.705ln(x).
          end{align*}






          share|cite|improve this answer












          The answer is neither! You need to take the logarithm of both sides:
          begin{align*}
          y&=e^{17.9348},x^{-2.705} \
          ln(y)&=lnleft(e^{17.9348},x^{-2.705}right) \
          ln(y)&=lnleft(e^{17.9348}right)+lnleft(x^{-2.705}right) \
          ln(y)&=17.9348-2.705ln(x).
          end{align*}







          share|cite|improve this answer












          share|cite|improve this answer



          share|cite|improve this answer










          answered 2 days ago









          Adrian Keister

          4,79851933




          4,79851933












          • ok, thank you so much!
            – user3063
            2 days ago


















          • ok, thank you so much!
            – user3063
            2 days ago
















          ok, thank you so much!
          – user3063
          2 days ago




          ok, thank you so much!
          – user3063
          2 days ago










          user3063 is a new contributor. Be nice, and check out our Code of Conduct.










          draft saved

          draft discarded


















          user3063 is a new contributor. Be nice, and check out our Code of Conduct.













          user3063 is a new contributor. Be nice, and check out our Code of Conduct.












          user3063 is a new contributor. Be nice, and check out our Code of Conduct.
















          Thanks for contributing an answer to Mathematics Stack Exchange!


          • Please be sure to answer the question. Provide details and share your research!

          But avoid



          • Asking for help, clarification, or responding to other answers.

          • Making statements based on opinion; back them up with references or personal experience.


          Use MathJax to format equations. MathJax reference.


          To learn more, see our tips on writing great answers.





          Some of your past answers have not been well-received, and you're in danger of being blocked from answering.


          Please pay close attention to the following guidance:


          • Please be sure to answer the question. Provide details and share your research!

          But avoid



          • Asking for help, clarification, or responding to other answers.

          • Making statements based on opinion; back them up with references or personal experience.


          To learn more, see our tips on writing great answers.




          draft saved


          draft discarded














          StackExchange.ready(
          function () {
          StackExchange.openid.initPostLogin('.new-post-login', 'https%3a%2f%2fmath.stackexchange.com%2fquestions%2f3060688%2flogarithmic-transformation-from-exponential-to-linear-equation%23new-answer', 'question_page');
          }
          );

          Post as a guest















          Required, but never shown





















































          Required, but never shown














          Required, but never shown












          Required, but never shown







          Required, but never shown

































          Required, but never shown














          Required, but never shown












          Required, but never shown







          Required, but never shown







          Popular posts from this blog

          An IMO inspired problem

          Management

          Investment